Awesome Roots Of Unity

Algebra Level 4

1 m α + 1 m α 2 + + 1 m α n \large{\dfrac{1}{m-\alpha}+\dfrac{1}{m-\alpha^2}+\cdots+\dfrac{1}{m-\alpha^n}}

Let P ( z ) = z n + 1 1 P(z)=z^{n+1}-1 have roots 1 , α , α 2 , , α n 1, \alpha, \alpha^2, \ldots, \alpha^n where α \alpha is a ( n + 1 ) th (n+1)^\text{th} root of unity and m m isn't a ( n + 1 ) th (n+1)^\text{th} root of unity.

Find the value of the expression above.

m n ( m n + m + 1 ) 1 ( m n + 1 1 ) ( m 1 ) \dfrac{m^n(mn+m+1)-1}{(m^{n+1}-1)(m-1)} m n ( m n n 1 ) + 1 ( m n + 1 1 ) ( m 1 ) \dfrac{m^n(mn-n-1)+1}{(m^{n+1}-1)(m-1)} m n ( m n m 1 ) + 1 ( m n + 1 1 ) ( m 1 ) \dfrac{m^n(mn-m-1)+1}{(m^{n+1}-1)(m-1)} m n ( m n + n + 1 ) 1 ( m n + 1 1 ) ( m 1 ) \dfrac{m^n(mn+n+1)-1}{(m^{n+1}-1)(m-1)}

This section requires Javascript.
You are seeing this because something didn't load right. We suggest you, (a) try refreshing the page, (b) enabling javascript if it is disabled on your browser and, finally, (c) loading the non-javascript version of this page . We're sorry about the hassle.

5 solutions

Sahil Bansal
Mar 21, 2016

P ( z ) P(z) can be expressed as follows:

z n + 1 1 = ( z 1 ) ( z α ) ( z α 2 ) ( z α 3 ) . . . ( z α n ) z^{n+1}-1=(z-1)(z-\alpha )(z-\alpha ^{2})(z-\alpha ^{3})...(z-\alpha ^{n})

Taking log both sides:

l o g ( z n + 1 1 ) = l o g ( z 1 ) + l o g ( z α ) + l o g ( z α 2 ) + . . . l o g ( z α n ) log(z^{n+1}-1)=log(z-1)+log(z-\alpha)+log(z-\alpha ^{2})+...log(z-\alpha ^{n})

Differentiating both sides with respect to z:

1 z n + 1 1 . ( n + 1 ) z n = 1 z 1 + 1 z α + 1 z α 2 + . . . + 1 z α n \frac{1}{z^{n+1}-1}.(n+1)z^{n}=\frac{1}{z-1}+\frac{1}{z-\alpha }+\frac{1}{z-\alpha^{2} }+...+\frac{1}{z-\alpha^{n} }

1 z n + 1 1 . ( n + 1 ) z n 1 z 1 = 1 z α + 1 z α 2 + . . . + 1 z α n \Rightarrow \frac{1}{z^{n+1}-1}.(n+1)z^{n}-\frac{1}{z-1}=\frac{1}{z-\alpha }+\frac{1}{z-\alpha^{2} }+...+\frac{1}{z-\alpha^{n} }

Replacing z by 'm' we get the required expression:

1 m α + 1 m α 2 + . . . + 1 m α n = 1 m n + 1 1 . ( n + 1 ) m n 1 m 1 \frac{1}{m-\alpha }+\frac{1}{m-\alpha^{2} }+...+\frac{1}{m-\alpha^{n} }= \frac{1}{m^{n+1}-1}.(n+1)m^{n}-\frac{1}{m-1}

= ( n + 1 ) m n ( m 1 ) ( m n + 1 1 ) ( m n + 1 1 ) ( m 1 ) =\frac{(n+1)m^{n}(m-1)-(m^{n+1}-1)}{(m^{n+1}-1)(m-1)}

On rearranging the terms of numerator after opening brackets:

1 m α + 1 m α 2 + . . . + 1 m α n = m n ( m n n 1 ) + 1 ( m n + 1 1 ) ( m 1 ) \frac{1}{m-\alpha }+\frac{1}{m-\alpha^{2} }+...+\frac{1}{m-\alpha^{n} }=\boxed{\frac{m^{n}(mn-n-1)+1}{(m^{n+1}-1)(m-1)}}

Did exactly the same way. Thumbs Up!

Miraj Shah - 5 years, 2 months ago
Otto Bretscher
Mar 21, 2016

It's good to have you back, Compañero Alan... I love your problems!

Awesome roots of unity indeed! The problem turns out to be fairly straightforward as long as we keep our m m 's and n n 's straight ;)

Make a substitution w = 1 m z w=\frac{1}{m-z} or z = m w 1 w z=\frac{mw-1}{w} . Now z n + 1 = 1 z^{n+1}=1 gives ( m w 1 ) n + 1 = w n + 1 (mw-1)^{n+1}=w^{n+1} or ( m n + 1 1 ) w n + 1 ( n + 1 ) m n w n + . . . = 0 (m^{n+1}-1)w^{n+1}-(n+1)m^nw^n+...=0 . By Viète, the sum over all the roots of unity α k \alpha^k for 0 k n 0\leq k \leq n is ( n + 1 ) m n m n + 1 1 \frac{(n+1)m^n}{m^{n+1}-1} . Subtracting 1 m 1 \frac{1}{m-1} for α 0 = 1 \alpha^0=1 gives m n ( m n n 1 ) + 1 ( m n + 1 1 ) ( m 1 ) \boxed{\frac{m^n(mn-n-1)+1}{(m^{n+1}-1)(m-1)}} .

Here we need to assume that m m isn't an ( n + 1 ) t h (n+1)^{th} root of unity.

Moderator note:

Nicely done. Usually for such problems, the correct substitution + use of Vieta's allows us to avoid tedious algebraic manipulation to obtain the expression. Of course, we may have to deal with an ugly expansion / collecting terms.

Ohh... I was so willing to write a solution for this problem... I was posting this particular solution only but I was stuck since I didn't subtracted α 0 \alpha^0 ( I almost checked my calculations 5-6 times but not getting the final answer )....... If this question had been under Calculus category I would already have posted the solution. ;-).. BTW I also tried using k = 1 n 1 m α k + 1 m α n + 1 k \sum_{k=1}^{n}\dfrac{1}{m-\alpha^k}+\dfrac{1}{m-\alpha^{n+1-k}} only to realise that it is beneficial under some special cases only.. Sorry for writing too long.. BTW (+1)...

Rishabh Jain - 5 years, 2 months ago

Log in to reply

I'm sorry my solution is so long ;)

Otto Bretscher - 5 years, 2 months ago

Awesome solution too! I've fixed the statement of the problem.

Alan Enrique Ontiveros Salazar - 5 years, 2 months ago
汶良 林
Mar 26, 2016

Akarsh Kumar Srit
Mar 21, 2016

Just consider value of n=2 it will convert into cube roots of unity and then solve

classic jee style mr.Taw

Zerocool 141 - 4 years, 8 months ago
Aakash Khandelwal
Mar 21, 2016

Assume P ( z ) = z n + z n 1 . . . . + z + 1 P(z) = z^{n} + z^{n-1} ....+z+1 = ( x α ) ( x α 2 ) . . . . ( x α n ) (x-\alpha)(x-\alpha^{2})....(x-\alpha^{n}) . Take natural logarithm on both sides of equation , and d i f f e r e n t i a t e differentiate . Use sum of n n terms if an GP S n S_n = a [ r n 1 / r 1 ] a[r^{n}- 1/r-1] , where a a is first term of a GP containing n n terms with a common ratio r r . Rearrange to get the desired answer.

0 pending reports

×

Problem Loading...

Note Loading...

Set Loading...